Đến nội dung

nam8298 nội dung

Có 158 mục bởi nam8298 (Tìm giới hạn từ 29-03-2020)



Sắp theo                Sắp xếp  

#459202 Chia tam giác đều cạnh $n$ thành $n^2$ tam giác đều cạnh...

Đã gửi bởi nam8298 on 22-10-2013 - 12:48 trong Tổ hợp và rời rạc

Bài 1 trong cuộc hội thảo cứ 10 người thì có đúng 1 người quen chung tìm số người quen lớn nhất của 1 người

Bài 2 Cho đa giác lồi n đỉnh sao cho không có 3 đường chéo nào đồng quy.tìm số miền do các đường chéo tạo nên

Bài 3 một tam giác đều n cạnh được chia làm $n^{2}$ tam giác đều cạnh 1 bằng các đường thẳng song song với các cạnh của nó .Hỏi có bao nhiêu tam giác đều được tạo thành

Bài 4 cho số nguyên $n\geq 2$ CMR trong mọi họ gồm ít nhất $2^{n-1}+1$ tập con không rỗng phân biệt của tập {1,2,3.....,n} đều tìm được 3 tập mà một trong chúng là hợp của 2 tập còn lại

 




#477414 Tìm GTNN của : A=$\frac{x+8}{\sqrt{x}...

Đã gửi bởi nam8298 on 15-01-2014 - 19:52 trong Bất đẳng thức và cực trị

A-4 =$\frac{(\sqrt{x}-2)^{2}}{\sqrt{x}+1}\geq 0$ nên GTNN A =4




#456633 Đề thi chọn đội tuyển HSG TP Hà Nội

Đã gửi bởi nam8298 on 10-10-2013 - 19:57 trong Thi HSG cấp Tỉnh, Thành phố. Olympic 30-4. Đề thi và kiểm tra đội tuyển các cấp.

KỲ THI CHỌN ĐỘI TUYỂN HSG THÀNH PHỐ
Năm học 2013-2014



đề thi HSG.jpg



#460344 Đề thi HSG lớp 12 tỉnh Vĩnh Phúc 2013-2014

Đã gửi bởi nam8298 on 27-10-2013 - 19:12 trong Thi HSG cấp Tỉnh, Thành phố. Olympic 30-4. Đề thi và kiểm tra đội tuyển các cấp.

xin lỗi mình đánh vội quá nên sai đề




#460039 Đề thi HSG lớp 12 tỉnh Vĩnh Phúc 2013-2014

Đã gửi bởi nam8298 on 26-10-2013 - 12:02 trong Thi HSG cấp Tỉnh, Thành phố. Olympic 30-4. Đề thi và kiểm tra đội tuyển các cấp.

1


Đăng lại đề @@
Bài 1. Cho trước số thực $a>0$ và dãy số thực $x_{n}$ xác định bởi $x_{1}$ =a và $x_{n+1}= \sqrt{17+16x_{n}}$ với mọi $n\geq 1$. Chứng minh rằng với mọi $a>0$ dãy $x_{n}$ có giới hạn khi $n\rightarrow$ dương vô cùng..Tìm giới hạn đó
Bài 2. Cho $3$ số $x,y,z$ không âm thỏa mãn $x^{2}+y^{2}+z^{2}= 1$ CMR $\sqrt{1-\frac{(x+y^{2})}{4}}+\sqrt{1-\frac{(y+z)^{2}}{4}}+\sqrt{1-\frac{(z+x)^{2}}{4}}\geq \sqrt{6}$
Bài 3. Tìm các số tự nhiên $x,y$ thỏa mãn phương trình ($(x^{2}+y)(y^{2}+x)= 2(x-y)^{3}$
Bài 4. Cho tam giác $ABC$ nhọn nội tiếp đường tròn $(O)$ vơí $AB<AC$ .Tiếp tuyến tại $A$ của $(O)$ cắt $BC$ tại $E$ . $D$ là điểm đối xứng của $A$ qua $O$,
a, Chứng minh rằng $AE$ song song với $CD$
b, Đường thẳng $BE$ cắt $AT$ tại $F$ .Giả sử đường tròn ngoại tiếp tam giác $AEF$ cắt $EO$ tại $G$ khác điểm $E$ .Chứng minh rằng tâm đường tròn nội tiếp tam giác $AGB$ nằm trên $(O)$
Bài 5. Một số nguyên dương $k$ được gọi là số đẹp nếu có thể phân hoạch tập hợp các số nguyên dương thành $k$ tập $A_{1},A_{2}....A_{k}$ sao cho với mỗi số nguyên dương $n\geq 15$ và với mọi i$\in (1;2;....:k)$ đều tồn tại 2 số thuộc $A_{i}$ có tổng là $n$
a, Chứng minh rằng $k=3$ là số đẹp
b. Chứng minh rằng với mọi $k\geq 4$ đều không đẹp.



#471620 Chứng minh đa thức f(x) chia hết cho đa thức g(x)

Đã gửi bởi nam8298 on 18-12-2013 - 20:24 trong Đại số

cái đa thức đầu tiên là $\frac{x^{80}-1}{x-1}$ .cái thứ 2 là $\frac{x^{20}-1}{x-1}$ .cái đầu rõ ràng chia hết cho cái sau




#456172 Cho x,y thỏa mãn $8x^2+y^2+\dfrac{1}{4x^2}=4...

Đã gửi bởi nam8298 on 08-10-2013 - 20:12 trong Bất đẳng thức và cực trị

công thức nghiệm bậc ba có trong NCPT 9 đấy bạn ơi




#466541 Chứng minh: A= $a^n+b^n+c^n+d^n$ là hợp số với mọi n tự nhiên.

Đã gửi bởi nam8298 on 24-11-2013 - 19:44 trong Số học

nếu a khác c

sau khi quy đồng ta đc ac =$b^{2}$

$b^{2}+a^{2}+c^{2}= a^{2}+c^{2}-ac= (a+c)^{2}-ac= (a+c)^{2}-b^{2}= (a+b+c)(a+c-b)$

nếu a+c-b =1 suy ra $ac=b^{2}=(a+c-1)^{2}$ hay $a^{2}+c^{2}+ac-2a-2c+1=0$ hay $(a-1)^{2}+(c-1)^{2}+ac-1=0$ suy ra ac =1 suy ra a=c=1 ( vô lí do a khác c)




#466421 Chứng minh: A= $a^n+b^n+c^n+d^n$ là hợp số với mọi n tự nhiên.

Đã gửi bởi nam8298 on 24-11-2013 - 09:33 trong Số học

bài 2 : phải có a khác c .nếu a=c thì có bộ thỏa mãn như a=c=2 .b=3 thì $a^{2}+b^{2}+c^{2}= 17$ là số nguyên tố




#463634 $n(n+1)(2n+1)\vdots 42$

Đã gửi bởi nam8298 on 11-11-2013 - 19:20 trong Đại số

bài 1 thay x =7k +3 ta đc (7k+3)(7k+4)(14k+7) cái này hiển nhiên là chia hết cho 42




#471617 Giải phương trình nghiệm nguyên dương : $(x^2+1)(y^2+1)+2(x-y)(1-xy)=4(...

Đã gửi bởi nam8298 on 18-12-2013 - 20:18 trong Số học

mình nhớ là bài này sau khi bung hết ra sẽ đc 1 cái phương trình tích .sau đó giải đc x,y




#472510 $A=a^3+b^3+c^3+4abc\leq \frac{9}{32}$

Đã gửi bởi nam8298 on 23-12-2013 - 19:50 trong Bất đẳng thức và cực trị

giả sử a là max {a,b,c}

BĐT cần chứng minh tương đương với $\frac{23}{32}+7abc\leq 3(ab+bc+ca)$

mặt khác ta có $(1-2a)(1-2b)(1-2c)\geq 0\Leftrightarrow 3(ab+bc+ca)\geq 6abc+\frac{3}{4}$

ta chứng minh 6abc+$6abc+\frac{3}{4}\geq 7abc+\frac{23}{32}\Leftrightarrow \frac{1}{32}\geq abc$

áp dụng AM-GM ta có abc $\leq a\frac{(b+c)^{2}}{4}= a\frac{(1-a)^{2}}{4}\leq \frac{1}{32}$




#466430 $\frac{a^2}{5a^2+(b+c)^2}+\frac{b^2...

Đã gửi bởi nam8298 on 24-11-2013 - 09:54 trong Bất đẳng thức và cực trị

Bài này hình như đâu chuân hoá dc đâu

 bài này chuẩn hóa đc mà bạn




#466275 $\frac{a^2}{5a^2+(b+c)^2}+\frac{b^2...

Đã gửi bởi nam8298 on 23-11-2013 - 19:22 trong Bất đẳng thức và cực trị

chuẩn hóa a+b+c =3 .ta chứng minh $\frac{a^{2}}{5a^{2}+(3-a)^{2}}\leq \frac{1}{3}+\frac{4}{9}(a-1)$ ( biến đổi tương đương )

tương tự cọng theo vế đc đpcm




#499256 Mâu thuẫn giữa 2 ĐHV

Đã gửi bởi nam8298 on 15-05-2014 - 20:48 trong Xử lí vi phạm - Tranh chấp - Khiếu nại

Có vẻ mâu thuẫn bắt đầu từ 27/4 khi toc ngan nhắc nhở buitudong1998




#462558 Chứng minh rằng nếu A là tích của n số nguyên tố đầu tiên thì: p+1 và p-1 khô...

Đã gửi bởi nam8298 on 06-11-2013 - 21:02 trong Số học

 theo mình p là tích của n số nguyên tố đầu tiên .nếu thế mình chứng minh thế này

      p chia hết cho 3 nên p-1 chia 3 dư 2 nên không là số chính phương

      giả sử p+1 là số chính phương ..đặt p+1 =$a^{2}$ suy ra p =(a-1)(a+1) ..do p chẵn nên a lẻ .do đó a-1 và a+1 chẵn suy ra (a-1)(a+1) chia hết cho 4 suy ra p hia hết cho 4 (vô lí)

Vậy p-1 và p+1 không là số chính phương




#484446 $\sum \sqrt{a+(b-c)^{2}}\geq \sq...

Đã gửi bởi nam8298 on 23-02-2014 - 20:16 trong Bất đẳng thức - Cực trị

bài này  bình phương xong dùng Cauchy-Schwazt .

pp làm là cách nâng lũy thừa và điều chỉnh hệ số.

nó tương tự bài bđt thi chọn đôi tuyển Vĩnh Phúc năm 2013-2014.  lời giải hơi dài nên giờ mình ko kịp đánh ra




#486476 $x^2+y^2+z^2\geq x^3+y^3+z^3$

Đã gửi bởi nam8298 on 12-03-2014 - 19:18 trong Bất đẳng thức và cực trị

do  -1 <= x ,y ,z <= 1 nên x^3 <= x^2 .

tương tự rồi cộng lại là đc.




#477583 Cho x, y, z là các số hữu tỉ dương thỏa mãn $x+\frac{1}...

Đã gửi bởi nam8298 on 16-01-2014 - 19:34 trong Bất đẳng thức và cực trị

Đặt xyz= t=$\frac{m}{n}$ với ( m,n ) =1

Ta có $\frac{t+1}{yz};\frac{t+1}{zx};\frac{t+1}{xy}$ là các số nguyên dương .suy ra $\frac{(t+1)^{3}}{t^{2}}$ là số nguyên dương.

hay $\frac{(m+n)^{3}}{m^{2}n}$ là số nguyên dương .

mà ( m,n ) =1 nên n =1 . do đó xyz =m là số nguyên dương .

$\frac{(m+1)^{3}}{m^{2}}$ là số nguyên nên m =1 .suy ra 2x ,2y ,2z là số nguyên dương

ta có 2x .2y .2z =8 .Từ đâytìm được các bộ (x,y,z )

thay vào biểu thức tìm max là được




#455709 Tìm điểm M sao cho MA+MB+MC+MD+ME đạt GTLN và GTNN

Đã gửi bởi nam8298 on 06-10-2013 - 20:01 trong Bất đẳng thức và cực trị

hạ MI,MJ,MK,MH,MO vuông góc với AB,BC,CD,DE,EA.ta có 2(MA+MB+MC+MD+ME) =$\sqrt{MI^{2}+AI^{^2}}$ +........sau đó áp dụng mincopski .thay (MI+MJ+MK+MH+MO)=2S(abcde)/(AB+BC+CD+DE+EA) rồi dùng AM-GM thì tìm được min.còn max thì mình không biết




#456892 Chứng minh

Đã gửi bởi nam8298 on 11-10-2013 - 20:40 trong Bất đẳng thức và cực trị

 ta có $2(a+b+c)-abc=a(2-bc)+2(b+c)\leq \sqrt{(a^{2}+(b+c)^{2})((2-bc)^{2}+4)}= \sqrt{(9+2bc)(b^{2}c^{2}-4bc+8)}$.đến đây biến đổi tương đương là đc




#477591 x,y,z>0,x+y+z=1.CMR: $\frac{x}{x^2+1}+...

Đã gửi bởi nam8298 on 16-01-2014 - 19:52 trong Bất đẳng thức và cực trị

2. nếu đề yêu cầu chứng minh $\geq 3$ :

 ta có $\frac{x+3}{(x+1)^{2}}\geq 1+\frac{3}{4}(x-1)$

chứng minh tương tự rồi cộng theo vế đc đpcm




#477589 x,y,z>0,x+y+z=1.CMR: $\frac{x}{x^2+1}+...

Đã gửi bởi nam8298 on 16-01-2014 - 19:47 trong Bất đẳng thức và cực trị

1. ta có $\frac{x}{x^{2}+1}\leq \frac{3}{10}+\frac{6}{25}(x-\frac{1}{3})$

tương tự cộng theo vế đc đpcm




#478376 $\sum \sqrt{a+(b-c)^{2}}\geq \sq...

Đã gửi bởi nam8298 on 21-01-2014 - 19:51 trong Bất đẳng thức - Cực trị

bình phương hai vế ta đc BĐT cần chứng minh tương đương với $3(ab+bc+ca)\leq \sum \sqrt{a^{2}+ab+ac+(b-c)^{2}}\sqrt{b^{2}+ba+bc+(c-a)^{2}}$

 áp dụng Cauchy -Schwazt ta có X= $\sqrt{a^{2}+ab+ac+(b-c)^{2}}\sqrt{b^{2}+ba+bc+(c-a)^{2}}= \sqrt{\sqrt{a(a+b+c)}^{2}+(b-c)^{2}}\sqrt{\sqrt{b(b+a+c)}^{2}+(c-a)^{2}}$ $\geq \left | (b-c)(c-a) \right |+\sqrt{ab}(a+b+c)$

làm tương tự rồi cộng lại ta cần chứng minh $\sum \left | (b-c)(c-a) \right |\geq 3(ab+bc+ca)-(a+b+c)(\sqrt{ab}+\sqrt{bc}+\sqrt{ca})$

do $\sum \left | (b-c)(c-a) \right |\geq (\sum a^{2})-ab-bc-ca$ nên ta cần chứng minh $(\sum a^{2})+(a+b+c)(\sqrt{ab}+\sqrt{bc}+\sqrt{ca})\geq 4(ab+bc+ca)$

có thể viết dưới dạng $\sum (x-y)^{2}xy+\sum x^{4}+xyz(x+y+z)\geq 2\sum x^{2}y^{2}$  (luôn đúng theo Schur )

Vậy BĐT đc chứng minh




#466490 Chứng minh rằng: Nếu $1+2^n+4^n$ là số nguyên tố thì tồn tại $...

Đã gửi bởi nam8298 on 24-11-2013 - 15:30 trong Số học

đặt n =$3^{k}m$ ( m không chia hết cho 3 )

nếu m =3l+1   suy ra $1+2^{n}+4^{n}$ =$a(a^{3l}-1)+a^{2}(a^{6l-1})+a^{2}+a+1$ chia hết cho a^{2}+a+1$ nên không là số nguyên tố

nếu m=3l+2    .làm tương tự ta đc $1+2^{n}+4^{n}$ chia hết cho a^{2}+a+1$ nên không là số nguyên tố

vậy n=$3^{k}$